Find the value of complex expression $left(fracsqrt3+i2right)^69$

The name of the pictureThe name of the pictureThe name of the pictureClash Royale CLAN TAG#URR8PPP











up vote
1
down vote

favorite
1












Find the value of




$$left(dfracsqrt3+i2right)^69.DeclareMathOperatorciscis$$




I tried to solve this complex expression by converting it into polar form.
I expressed it in polar form $rcis(t)$ from rectangular form $x+iy$ where $cis(t) = cos(t) + isin(t)$.
But I am unable to solve further due to the exponent of 69!







share|cite|improve this question


























    up vote
    1
    down vote

    favorite
    1












    Find the value of




    $$left(dfracsqrt3+i2right)^69.DeclareMathOperatorciscis$$




    I tried to solve this complex expression by converting it into polar form.
    I expressed it in polar form $rcis(t)$ from rectangular form $x+iy$ where $cis(t) = cos(t) + isin(t)$.
    But I am unable to solve further due to the exponent of 69!







    share|cite|improve this question
























      up vote
      1
      down vote

      favorite
      1









      up vote
      1
      down vote

      favorite
      1






      1





      Find the value of




      $$left(dfracsqrt3+i2right)^69.DeclareMathOperatorciscis$$




      I tried to solve this complex expression by converting it into polar form.
      I expressed it in polar form $rcis(t)$ from rectangular form $x+iy$ where $cis(t) = cos(t) + isin(t)$.
      But I am unable to solve further due to the exponent of 69!







      share|cite|improve this question














      Find the value of




      $$left(dfracsqrt3+i2right)^69.DeclareMathOperatorciscis$$




      I tried to solve this complex expression by converting it into polar form.
      I expressed it in polar form $rcis(t)$ from rectangular form $x+iy$ where $cis(t) = cos(t) + isin(t)$.
      But I am unable to solve further due to the exponent of 69!









      share|cite|improve this question













      share|cite|improve this question




      share|cite|improve this question








      edited Aug 17 at 4:28









      Theo Bendit

      12.2k1844




      12.2k1844










      asked Aug 17 at 2:28









      Shubh Khandelwal

      675229




      675229




















          4 Answers
          4






          active

          oldest

          votes

















          up vote
          3
          down vote



          accepted










          $$left(dfracsqrt3+i2right)^69=left(dfracsqrt32+dfrac12iright)^69=(cosdfracpi6+isindfracpi6)^69=cosdfrac69pi6+isindfrac69pi6=-i$$
          by De Moivre's formula.






          share|cite|improve this answer






















          • Thanks Nosrati, I didn't know about De Moivrer's formula.
            – Shubh Khandelwal
            Aug 17 at 2:45










          • You are welcome dear!
            – Nosrati
            Aug 17 at 2:46










          • @Shubh Khandelwal Basically complex multiplication has the rule that the norm of the product is the product of norms, and the argument of product is the sum of arguments.
            – MonkeyKing
            Aug 17 at 3:38

















          up vote
          3
          down vote













          Converting to polar coordinates is probably the best way to go.



          $$r = sqrtleft(fracsqrt32right)^2+left(frac12right)^2=1$$



          $$theta = arctan(1/sqrt3)=fracpi6$$



          Then $$(re^theta i )^69=e^frac69pi6i=e^10pi i+frac3pi2i=e^frac3pi2i=-i$$






          share|cite|improve this answer




















          • this is same as my answer which I published minutes ago
            – James
            Aug 17 at 2:52






          • 2




            @James Yes, and? Great minds think alike, no? That being said, this answer is not the same answer that you published. It is more detailed in terms of explaining the conversion from rectangular to exponential form. This is neither good nor bad, just different.
            – Xander Henderson
            Aug 17 at 3:25

















          up vote
          0
          down vote













          $$fracsqrt3+i2=e^ifracpi6$$



          $$big(fracsqrt3+i2big)^69=e^ifracpi6cdot69=e^i(11pi+fracpi2)=e^i(12pi-fracpi2)=e^-ifracpi2=-i$$



          I used the fact that $e^i(2npi)=1$ where $nin Z$






          share|cite|improve this answer





























            up vote
            0
            down vote













            Bruteforcing:
            $$beginalignleft(dfracsqrt3+i2right)^69&=left[left(dfracsqrt3+i2right)^2right]^34cdot fracsqrt3+i2=\
            &=left[left(frac1+sqrt3i2right)^2right]^17cdot fracsqrt3+i2=\
            &=left[left(frac-1+sqrt3i2right)^2right]^8cdot frac-1+sqrt3i2cdot fracsqrt3+i2=\
            &=left[left(frac-1-sqrt3i2right)^2right]^4cdot frac-sqrt3+i2=\
            &=left[left(frac-1+sqrt3i2right)^2right]^2cdot frac-sqrt3+i2=\
            &=left[frac-1-sqrt3i2right]^2cdot frac-sqrt3+i2=\
            &=frac-1+sqrt3i2cdot frac-sqrt3+i2=\
            &=-i.endalign$$
            Alternatively:
            $$E=left(fracsqrt3+i2right)^3cdot 23=left(frac3sqrt3+9i-3sqrt3-i8right)^23=i^23=(i^2)^11cdot i=-i.$$






            share|cite|improve this answer






















            • Bruteforcing is not the best way,but thank you.
              – Shubh Khandelwal
              Aug 24 at 4:11










            Your Answer




            StackExchange.ifUsing("editor", function ()
            return StackExchange.using("mathjaxEditing", function ()
            StackExchange.MarkdownEditor.creationCallbacks.add(function (editor, postfix)
            StackExchange.mathjaxEditing.prepareWmdForMathJax(editor, postfix, [["$", "$"], ["\\(","\\)"]]);
            );
            );
            , "mathjax-editing");

            StackExchange.ready(function()
            var channelOptions =
            tags: "".split(" "),
            id: "69"
            ;
            initTagRenderer("".split(" "), "".split(" "), channelOptions);

            StackExchange.using("externalEditor", function()
            // Have to fire editor after snippets, if snippets enabled
            if (StackExchange.settings.snippets.snippetsEnabled)
            StackExchange.using("snippets", function()
            createEditor();
            );

            else
            createEditor();

            );

            function createEditor()
            StackExchange.prepareEditor(
            heartbeatType: 'answer',
            convertImagesToLinks: true,
            noModals: false,
            showLowRepImageUploadWarning: true,
            reputationToPostImages: 10,
            bindNavPrevention: true,
            postfix: "",
            noCode: true, onDemand: true,
            discardSelector: ".discard-answer"
            ,immediatelyShowMarkdownHelp:true
            );



            );








             

            draft saved


            draft discarded


















            StackExchange.ready(
            function ()
            StackExchange.openid.initPostLogin('.new-post-login', 'https%3a%2f%2fmath.stackexchange.com%2fquestions%2f2885339%2ffind-the-value-of-complex-expression-left-frac-sqrt3i2-right69%23new-answer', 'question_page');

            );

            Post as a guest






























            4 Answers
            4






            active

            oldest

            votes








            4 Answers
            4






            active

            oldest

            votes









            active

            oldest

            votes






            active

            oldest

            votes








            up vote
            3
            down vote



            accepted










            $$left(dfracsqrt3+i2right)^69=left(dfracsqrt32+dfrac12iright)^69=(cosdfracpi6+isindfracpi6)^69=cosdfrac69pi6+isindfrac69pi6=-i$$
            by De Moivre's formula.






            share|cite|improve this answer






















            • Thanks Nosrati, I didn't know about De Moivrer's formula.
              – Shubh Khandelwal
              Aug 17 at 2:45










            • You are welcome dear!
              – Nosrati
              Aug 17 at 2:46










            • @Shubh Khandelwal Basically complex multiplication has the rule that the norm of the product is the product of norms, and the argument of product is the sum of arguments.
              – MonkeyKing
              Aug 17 at 3:38














            up vote
            3
            down vote



            accepted










            $$left(dfracsqrt3+i2right)^69=left(dfracsqrt32+dfrac12iright)^69=(cosdfracpi6+isindfracpi6)^69=cosdfrac69pi6+isindfrac69pi6=-i$$
            by De Moivre's formula.






            share|cite|improve this answer






















            • Thanks Nosrati, I didn't know about De Moivrer's formula.
              – Shubh Khandelwal
              Aug 17 at 2:45










            • You are welcome dear!
              – Nosrati
              Aug 17 at 2:46










            • @Shubh Khandelwal Basically complex multiplication has the rule that the norm of the product is the product of norms, and the argument of product is the sum of arguments.
              – MonkeyKing
              Aug 17 at 3:38












            up vote
            3
            down vote



            accepted







            up vote
            3
            down vote



            accepted






            $$left(dfracsqrt3+i2right)^69=left(dfracsqrt32+dfrac12iright)^69=(cosdfracpi6+isindfracpi6)^69=cosdfrac69pi6+isindfrac69pi6=-i$$
            by De Moivre's formula.






            share|cite|improve this answer














            $$left(dfracsqrt3+i2right)^69=left(dfracsqrt32+dfrac12iright)^69=(cosdfracpi6+isindfracpi6)^69=cosdfrac69pi6+isindfrac69pi6=-i$$
            by De Moivre's formula.







            share|cite|improve this answer














            share|cite|improve this answer



            share|cite|improve this answer








            edited Aug 17 at 13:56









            Akiva Weinberger

            13.2k12161




            13.2k12161










            answered Aug 17 at 2:33









            Nosrati

            20.7k41644




            20.7k41644











            • Thanks Nosrati, I didn't know about De Moivrer's formula.
              – Shubh Khandelwal
              Aug 17 at 2:45










            • You are welcome dear!
              – Nosrati
              Aug 17 at 2:46










            • @Shubh Khandelwal Basically complex multiplication has the rule that the norm of the product is the product of norms, and the argument of product is the sum of arguments.
              – MonkeyKing
              Aug 17 at 3:38
















            • Thanks Nosrati, I didn't know about De Moivrer's formula.
              – Shubh Khandelwal
              Aug 17 at 2:45










            • You are welcome dear!
              – Nosrati
              Aug 17 at 2:46










            • @Shubh Khandelwal Basically complex multiplication has the rule that the norm of the product is the product of norms, and the argument of product is the sum of arguments.
              – MonkeyKing
              Aug 17 at 3:38















            Thanks Nosrati, I didn't know about De Moivrer's formula.
            – Shubh Khandelwal
            Aug 17 at 2:45




            Thanks Nosrati, I didn't know about De Moivrer's formula.
            – Shubh Khandelwal
            Aug 17 at 2:45












            You are welcome dear!
            – Nosrati
            Aug 17 at 2:46




            You are welcome dear!
            – Nosrati
            Aug 17 at 2:46












            @Shubh Khandelwal Basically complex multiplication has the rule that the norm of the product is the product of norms, and the argument of product is the sum of arguments.
            – MonkeyKing
            Aug 17 at 3:38




            @Shubh Khandelwal Basically complex multiplication has the rule that the norm of the product is the product of norms, and the argument of product is the sum of arguments.
            – MonkeyKing
            Aug 17 at 3:38










            up vote
            3
            down vote













            Converting to polar coordinates is probably the best way to go.



            $$r = sqrtleft(fracsqrt32right)^2+left(frac12right)^2=1$$



            $$theta = arctan(1/sqrt3)=fracpi6$$



            Then $$(re^theta i )^69=e^frac69pi6i=e^10pi i+frac3pi2i=e^frac3pi2i=-i$$






            share|cite|improve this answer




















            • this is same as my answer which I published minutes ago
              – James
              Aug 17 at 2:52






            • 2




              @James Yes, and? Great minds think alike, no? That being said, this answer is not the same answer that you published. It is more detailed in terms of explaining the conversion from rectangular to exponential form. This is neither good nor bad, just different.
              – Xander Henderson
              Aug 17 at 3:25














            up vote
            3
            down vote













            Converting to polar coordinates is probably the best way to go.



            $$r = sqrtleft(fracsqrt32right)^2+left(frac12right)^2=1$$



            $$theta = arctan(1/sqrt3)=fracpi6$$



            Then $$(re^theta i )^69=e^frac69pi6i=e^10pi i+frac3pi2i=e^frac3pi2i=-i$$






            share|cite|improve this answer




















            • this is same as my answer which I published minutes ago
              – James
              Aug 17 at 2:52






            • 2




              @James Yes, and? Great minds think alike, no? That being said, this answer is not the same answer that you published. It is more detailed in terms of explaining the conversion from rectangular to exponential form. This is neither good nor bad, just different.
              – Xander Henderson
              Aug 17 at 3:25












            up vote
            3
            down vote










            up vote
            3
            down vote









            Converting to polar coordinates is probably the best way to go.



            $$r = sqrtleft(fracsqrt32right)^2+left(frac12right)^2=1$$



            $$theta = arctan(1/sqrt3)=fracpi6$$



            Then $$(re^theta i )^69=e^frac69pi6i=e^10pi i+frac3pi2i=e^frac3pi2i=-i$$






            share|cite|improve this answer












            Converting to polar coordinates is probably the best way to go.



            $$r = sqrtleft(fracsqrt32right)^2+left(frac12right)^2=1$$



            $$theta = arctan(1/sqrt3)=fracpi6$$



            Then $$(re^theta i )^69=e^frac69pi6i=e^10pi i+frac3pi2i=e^frac3pi2i=-i$$







            share|cite|improve this answer












            share|cite|improve this answer



            share|cite|improve this answer










            answered Aug 17 at 2:48









            rikhavshah

            947212




            947212











            • this is same as my answer which I published minutes ago
              – James
              Aug 17 at 2:52






            • 2




              @James Yes, and? Great minds think alike, no? That being said, this answer is not the same answer that you published. It is more detailed in terms of explaining the conversion from rectangular to exponential form. This is neither good nor bad, just different.
              – Xander Henderson
              Aug 17 at 3:25
















            • this is same as my answer which I published minutes ago
              – James
              Aug 17 at 2:52






            • 2




              @James Yes, and? Great minds think alike, no? That being said, this answer is not the same answer that you published. It is more detailed in terms of explaining the conversion from rectangular to exponential form. This is neither good nor bad, just different.
              – Xander Henderson
              Aug 17 at 3:25















            this is same as my answer which I published minutes ago
            – James
            Aug 17 at 2:52




            this is same as my answer which I published minutes ago
            – James
            Aug 17 at 2:52




            2




            2




            @James Yes, and? Great minds think alike, no? That being said, this answer is not the same answer that you published. It is more detailed in terms of explaining the conversion from rectangular to exponential form. This is neither good nor bad, just different.
            – Xander Henderson
            Aug 17 at 3:25




            @James Yes, and? Great minds think alike, no? That being said, this answer is not the same answer that you published. It is more detailed in terms of explaining the conversion from rectangular to exponential form. This is neither good nor bad, just different.
            – Xander Henderson
            Aug 17 at 3:25










            up vote
            0
            down vote













            $$fracsqrt3+i2=e^ifracpi6$$



            $$big(fracsqrt3+i2big)^69=e^ifracpi6cdot69=e^i(11pi+fracpi2)=e^i(12pi-fracpi2)=e^-ifracpi2=-i$$



            I used the fact that $e^i(2npi)=1$ where $nin Z$






            share|cite|improve this answer


























              up vote
              0
              down vote













              $$fracsqrt3+i2=e^ifracpi6$$



              $$big(fracsqrt3+i2big)^69=e^ifracpi6cdot69=e^i(11pi+fracpi2)=e^i(12pi-fracpi2)=e^-ifracpi2=-i$$



              I used the fact that $e^i(2npi)=1$ where $nin Z$






              share|cite|improve this answer
























                up vote
                0
                down vote










                up vote
                0
                down vote









                $$fracsqrt3+i2=e^ifracpi6$$



                $$big(fracsqrt3+i2big)^69=e^ifracpi6cdot69=e^i(11pi+fracpi2)=e^i(12pi-fracpi2)=e^-ifracpi2=-i$$



                I used the fact that $e^i(2npi)=1$ where $nin Z$






                share|cite|improve this answer














                $$fracsqrt3+i2=e^ifracpi6$$



                $$big(fracsqrt3+i2big)^69=e^ifracpi6cdot69=e^i(11pi+fracpi2)=e^i(12pi-fracpi2)=e^-ifracpi2=-i$$



                I used the fact that $e^i(2npi)=1$ where $nin Z$







                share|cite|improve this answer














                share|cite|improve this answer



                share|cite|improve this answer








                edited Aug 17 at 2:44

























                answered Aug 17 at 2:34









                James

                2,083619




                2,083619




















                    up vote
                    0
                    down vote













                    Bruteforcing:
                    $$beginalignleft(dfracsqrt3+i2right)^69&=left[left(dfracsqrt3+i2right)^2right]^34cdot fracsqrt3+i2=\
                    &=left[left(frac1+sqrt3i2right)^2right]^17cdot fracsqrt3+i2=\
                    &=left[left(frac-1+sqrt3i2right)^2right]^8cdot frac-1+sqrt3i2cdot fracsqrt3+i2=\
                    &=left[left(frac-1-sqrt3i2right)^2right]^4cdot frac-sqrt3+i2=\
                    &=left[left(frac-1+sqrt3i2right)^2right]^2cdot frac-sqrt3+i2=\
                    &=left[frac-1-sqrt3i2right]^2cdot frac-sqrt3+i2=\
                    &=frac-1+sqrt3i2cdot frac-sqrt3+i2=\
                    &=-i.endalign$$
                    Alternatively:
                    $$E=left(fracsqrt3+i2right)^3cdot 23=left(frac3sqrt3+9i-3sqrt3-i8right)^23=i^23=(i^2)^11cdot i=-i.$$






                    share|cite|improve this answer






















                    • Bruteforcing is not the best way,but thank you.
                      – Shubh Khandelwal
                      Aug 24 at 4:11














                    up vote
                    0
                    down vote













                    Bruteforcing:
                    $$beginalignleft(dfracsqrt3+i2right)^69&=left[left(dfracsqrt3+i2right)^2right]^34cdot fracsqrt3+i2=\
                    &=left[left(frac1+sqrt3i2right)^2right]^17cdot fracsqrt3+i2=\
                    &=left[left(frac-1+sqrt3i2right)^2right]^8cdot frac-1+sqrt3i2cdot fracsqrt3+i2=\
                    &=left[left(frac-1-sqrt3i2right)^2right]^4cdot frac-sqrt3+i2=\
                    &=left[left(frac-1+sqrt3i2right)^2right]^2cdot frac-sqrt3+i2=\
                    &=left[frac-1-sqrt3i2right]^2cdot frac-sqrt3+i2=\
                    &=frac-1+sqrt3i2cdot frac-sqrt3+i2=\
                    &=-i.endalign$$
                    Alternatively:
                    $$E=left(fracsqrt3+i2right)^3cdot 23=left(frac3sqrt3+9i-3sqrt3-i8right)^23=i^23=(i^2)^11cdot i=-i.$$






                    share|cite|improve this answer






















                    • Bruteforcing is not the best way,but thank you.
                      – Shubh Khandelwal
                      Aug 24 at 4:11












                    up vote
                    0
                    down vote










                    up vote
                    0
                    down vote









                    Bruteforcing:
                    $$beginalignleft(dfracsqrt3+i2right)^69&=left[left(dfracsqrt3+i2right)^2right]^34cdot fracsqrt3+i2=\
                    &=left[left(frac1+sqrt3i2right)^2right]^17cdot fracsqrt3+i2=\
                    &=left[left(frac-1+sqrt3i2right)^2right]^8cdot frac-1+sqrt3i2cdot fracsqrt3+i2=\
                    &=left[left(frac-1-sqrt3i2right)^2right]^4cdot frac-sqrt3+i2=\
                    &=left[left(frac-1+sqrt3i2right)^2right]^2cdot frac-sqrt3+i2=\
                    &=left[frac-1-sqrt3i2right]^2cdot frac-sqrt3+i2=\
                    &=frac-1+sqrt3i2cdot frac-sqrt3+i2=\
                    &=-i.endalign$$
                    Alternatively:
                    $$E=left(fracsqrt3+i2right)^3cdot 23=left(frac3sqrt3+9i-3sqrt3-i8right)^23=i^23=(i^2)^11cdot i=-i.$$






                    share|cite|improve this answer














                    Bruteforcing:
                    $$beginalignleft(dfracsqrt3+i2right)^69&=left[left(dfracsqrt3+i2right)^2right]^34cdot fracsqrt3+i2=\
                    &=left[left(frac1+sqrt3i2right)^2right]^17cdot fracsqrt3+i2=\
                    &=left[left(frac-1+sqrt3i2right)^2right]^8cdot frac-1+sqrt3i2cdot fracsqrt3+i2=\
                    &=left[left(frac-1-sqrt3i2right)^2right]^4cdot frac-sqrt3+i2=\
                    &=left[left(frac-1+sqrt3i2right)^2right]^2cdot frac-sqrt3+i2=\
                    &=left[frac-1-sqrt3i2right]^2cdot frac-sqrt3+i2=\
                    &=frac-1+sqrt3i2cdot frac-sqrt3+i2=\
                    &=-i.endalign$$
                    Alternatively:
                    $$E=left(fracsqrt3+i2right)^3cdot 23=left(frac3sqrt3+9i-3sqrt3-i8right)^23=i^23=(i^2)^11cdot i=-i.$$







                    share|cite|improve this answer














                    share|cite|improve this answer



                    share|cite|improve this answer








                    edited Aug 24 at 16:13

























                    answered Aug 17 at 3:46









                    farruhota

                    14k2632




                    14k2632











                    • Bruteforcing is not the best way,but thank you.
                      – Shubh Khandelwal
                      Aug 24 at 4:11
















                    • Bruteforcing is not the best way,but thank you.
                      – Shubh Khandelwal
                      Aug 24 at 4:11















                    Bruteforcing is not the best way,but thank you.
                    – Shubh Khandelwal
                    Aug 24 at 4:11




                    Bruteforcing is not the best way,but thank you.
                    – Shubh Khandelwal
                    Aug 24 at 4:11












                     

                    draft saved


                    draft discarded


























                     


                    draft saved


                    draft discarded














                    StackExchange.ready(
                    function ()
                    StackExchange.openid.initPostLogin('.new-post-login', 'https%3a%2f%2fmath.stackexchange.com%2fquestions%2f2885339%2ffind-the-value-of-complex-expression-left-frac-sqrt3i2-right69%23new-answer', 'question_page');

                    );

                    Post as a guest













































































                    這個網誌中的熱門文章

                    Is there any way to eliminate the singular point to solve this integral by hand or by approximations?

                    Why am i infinitely getting the same tweet with the Twitter Search API?

                    Carbon dioxide